Quantcast
  • Register
PhysicsOverflow is a next-generation academic platform for physicists and astronomers, including a community peer review system and a postgraduate-level discussion forum analogous to MathOverflow.

Welcome to PhysicsOverflow! PhysicsOverflow is an open platform for community peer review and graduate-level Physics discussion.

Please help promote PhysicsOverflow ads elsewhere if you like it.

News

PO is now at the Physics Department of Bielefeld University!

New printer friendly PO pages!

Migration to Bielefeld University was successful!

Please vote for this year's PhysicsOverflow ads!

Please do help out in categorising submissions. Submit a paper to PhysicsOverflow!

... see more

Tools for paper authors

Submit paper
Claim Paper Authorship

Tools for SE users

Search User
Reclaim SE Account
Request Account Merger
Nativise imported posts
Claim post (deleted users)
Import SE post

Users whose questions have been imported from Physics Stack Exchange, Theoretical Physics Stack Exchange, or any other Stack Exchange site are kindly requested to reclaim their account and not to register as a new user.

Public \(\beta\) tools

Report a bug with a feature
Request a new functionality
404 page design
Send feedback

Attributions

(propose a free ad)

Site Statistics

205 submissions , 163 unreviewed
5,047 questions , 2,200 unanswered
5,345 answers , 22,709 comments
1,470 users with positive rep
816 active unimported users
More ...

  Källén–Lehmann spectral representation for massless particle?

+ 6 like - 0 dislike
2003 views

Is it possible to write down a KL-like formula for massless particles (in particular, the photon)?

The usual proof of the theorem assumes (see http://www.thphys.uni-heidelberg.de/~weigand/QFT1-13-14/SkriptQFT1.pdf, page 44) that we can boost to a frame where the particle is at rest, and I couldn't find any proof that won't do this.

Details

Write the spectrum of the Hamiltonian as $$ H|p,\lambda\rangle=E_p(\lambda)|p,\lambda\rangle $$ $$ \boldsymbol P|p,\lambda\rangle=\boldsymbol p|p,\lambda\rangle $$ where $\lambda$ is any set of parameters that is needed to distinguish different states with the same energy-momentum.

Write the completeness relation of the eigenstates as $$ |\Omega\rangle\langle\Omega|+\sum_\lambda\int \widetilde{\mathrm d k}\ |p,\lambda\rangle\langle p,\lambda|=\mathbb I $$ and the exact propagator as $$ \boldsymbol\Delta(x,y)=\langle \Omega|T\ \phi(x)\phi(y)|\Omega\rangle $$

Insert the identity between the fields: $$ \boldsymbol\Delta(x,y)=\sum_\lambda\int \widetilde{\mathrm d k}\ \langle \Omega|\phi(x)|p,\lambda\rangle\langle p,\lambda|\phi(y)|\Omega\rangle $$ where I take the fields to have null vev.

Next, write $\phi(x)=e^{iPx}\phi(0)e^{-iPx}$, and boost to a frame where $p=(m,0,0,0)$, to get $\langle \Omega|\phi(x)|p,\lambda\rangle=\langle \Omega|\phi(0)|0,\lambda\rangle e^{-ipx}$. From this, the KL theorem follows.

Is it possible to circumvent this boost to get an alternative proof of the theorem? Or maybe the theorem is false for massless particles?

EDIT: Partial solution

See Wikipedia: https://en.wikipedia.org/wiki/Mass_gap

It seems that in general KL is false for gauge theories. In the case of the photon, the theorem is true, but it has to be checked explicitly. I don't know if this remains true in QCD.

Can anyone confirm this? Is the mass-gap problem related to my question? Is this (partial) solution correct?

This post imported from StackExchange Physics at 2016-05-31 07:23 (UTC), posted by SE-user AccidentalFourierTransform
asked Sep 27, 2015 in Theoretical Physics by AccidentalFourierTransform (480 points) [ no revision ]
Why do you actually need to boost at all? As far as I understand it's not actually need it and I think the proof goes through with $\langle\Omega|\phi(0)|p,\lambda\rangle$ as well. See e.g. chapter 10.7 in Weinberg volume I of QFT.

This post imported from StackExchange Physics at 2016-05-31 07:23 (UTC), posted by SE-user TwoBs

1 Answer

+ 5 like - 0 dislike

Källén-Lehmann representation is just a way to expand in the momentum basis the two point correlation function of a local operator $\hat{O}(x)$, it holds true for massive and massless theories alike except for non Abelian gauge theory in which the situation is a bit more complicated. Let's demonstrate the K-L formula with a more general proof: let's start with the one particle state

$$F(p)=\langle \Omega|\phi(0)|p,\lambda\rangle$$

which we'll see doesn't depend at all on $p$ . To be able to prove this result you just need to know how a scalar field transforms. Let $U(\Lambda)$ be an unitary representation of the Lorentz group, then:

$$U^{\dagger}(\Lambda)\phi(0)U(\Lambda)=\phi(0)$$

and the single particle state transforms like:

$$|\Lambda p,\lambda\rangle=U(\Lambda)| p ,\lambda\rangle$$

Thus, you can write

$$F(\Lambda p)=\langle \Omega|\phi(0)|\Lambda p,\lambda\rangle=\langle \Omega|\phi(0)U(\lambda)| p,\lambda\rangle=\langle \Omega|U(\Lambda)U^{\dagger}(\Lambda)\phi(0)U(\lambda)| p,\lambda\rangle$$

but $U^{\dagger}(\Lambda)\phi(0)U(\lambda)=\phi(0)$ and by definition $| \Omega\rangle=U(\Lambda)| \Omega\rangle$ then you have

$$F(\Lambda p)=F(p)$$

which means that $F=F(p^2)$ is a function of a Lorentz scalar. Now, you can write $p^2=m^2$ ($m^2$ might be zero; the important thing is $p^2=\text{const.}$) so you have that

$$F=F(m^2)= \sqrt{Z}$$

Note that all steps so far have been carried out without assuming the spectrum of the theory has any mass gap, indeed the states we boosted are completely arbitrary and not in some rest frame.

For multi-particle states you can just use

$$\langle O(x)O(0)\rangle=\int_{0}^{\infty} d\mu^2 \rho(\mu^2) \ i\Delta_F(x,\mu^2) \tag 1$$

where the propagator has a fictitious mass $\mu^2$, in fact in this case the continuum of states starts right in $p=0$ and there is no forbidden gap between the one particle state and the multi-particle states.

To demonstrate the general formula one starts from: (from now on, I will forget about time ordering and stuff and just consider $\Delta_F^+=\Delta_F$ just to skip unnecessary passages, also I'll assume from the start that the theory is massless!)

$$\langle O(x)O(0)\rangle=\langle 0|O(x)\sum_n |n,\lambda\rangle\langle n,\lambda|O(0)|0\rangle=\sum_n |\langle 0|O(0) |n,\lambda\rangle|^2 e^{-iP_n \cdot x}$$

where we define the identity as

$$\mathbb{I}=|0\rangle\langle 0|+\int \frac{d^3p}{(2\pi)^3}\frac{1}{2|p|}\sum_{\lambda}|p,\lambda\rangle\langle p,\lambda|+...$$

and have chosen the covariant normalization of states (i.e. the measue $d\sigma=\frac{d^3p}{(2\pi)^3}\frac{1}{2|p|}$ is lorentz invariant and states transform as above) and $P_n$ is the momentum associated with the n-th particle state. Now let's write

$$\langle O(x)O(0)\rangle=\int d^4q \ e^{-iqx}\ \sum_n |\langle 0|O(0) |n,\lambda\rangle|^2 \delta^4(q-P_n)$$

And introduce

$$\rho(q)= \sum_n |\langle 0|O(0) |n,\lambda\rangle|^2 \delta^4(q-P_n)$$

It is easy to prove that

$$\rho(\Lambda q)=\rho(q) \rightarrow \rho=\rho(q^2)$$

by using the fact that the $\delta$ is invariant and that the identity $\mathbb I$ doesn't care if the states are boosted or not, i.e. $\mathbb I= \sum_n |n,\lambda\rangle\langle n,\lambda|=U(\Lambda)\mathbb I U^{\dagger}(\Lambda)=\sum_n |\Lambda n,\lambda\rangle\langle \Lambda n,\lambda|=\mathbb I$ where we have used the unitarity of the Lorentz transformation representation $U \ U^{\dagger}=\mathbb I$) moreover its support is contained in the spectrum of the theory so

$$\rho(q)=0 \qquad \text{ if } \qquad q^0<0\\ \rho(q)=0 \qquad \text{ if } \qquad q^2<0$$

Let's reparametrize $\rho$ as follows: $\rho(q^2)\rightarrow \frac{\theta(q^0)}{(2\pi)^3}\rho(q^2)$ in order to make obvious the second condition on $\rho$. Now we can write

$$\langle O(x)O(0)\rangle=\int \frac{d^4q}{(2\pi)^3} \theta(q^0)\rho(q^2) \ e^{-iqx}=\int d\mu^2\int \frac{d^4q}{(2\pi)^3} \theta(q^0)\rho(q^2) \ e^{-iqx} \delta(q^2-\mu^2)$$

where $\mu$ is really just an arbitrary parameter. Now we can see that

$$i\Delta_F(x,\mu^2)=\int \frac{d^4q}{(2\pi)^3} \theta(q^0)\ e^{-iqx} \delta(q^2-\mu^2)=\int \frac{d^3q}{(2\pi)^3}\frac{1}{2\sqrt{|q|^2+\mu^2}} e^{-ixq}$$

It's clear now that we can write the two point function as in $(1)$, to make manifest the presence of the one particle state the decomposition of $\rho$ will be

$$\rho=\frac{Z}{(2\pi)^3}\delta(q^2)+\rho_{M}$$

(where $M$ stands for multi-particle states) then we will have

$$\langle O(x)O(0)\rangle=Z \ i\Delta_F(x,0)+\int_{0}^{\infty} d\mu^2 \rho_{M}(\mu^2) \ i\Delta_F(x,\mu^2)$$

where

$$i\Delta_F(x,0)=\int \frac{d^3p}{(2\pi)^3}\frac{1}{2|p|} e^{-ixp}$$

As far as i know the same argument and proof (with some tweaks to take in account the polarization degrees of freedom) goes for the photon two point function. For non-Abelian gauge theories it's not possible to have a well-defined $\rho$ by itself because it is necessary to take into account the ghost fields which have the role of canceling out unphysical degrees of freedom of the gluon propagator. That's why i think it's not very practical to use this formalism in the context of non-Abelian gauge fields where it's easier to operate from a path integral approach.

EDIT (answer to comment):

The K-L decomposition is done using a basis of the momentum operator representing the intermediate states between the initial and final position of the particle thus when you consider the one particle contribution it means it is simply the free particle propagating without any intermediate states (no loops if you look at it perturbatively). Then the only changes from the free theory are the physical mass being influenced by the interaction and the renormalization constant Z which means that the probability of having a one particle contribution is no longer 1 as it was in the free theory where the field $\phi$ could just produce one particle states. This also means that being a particle propagating by itself with no force acting upon it, when you consider the one particle contribution it will have a well defined energy $p^0=E(\textbf{p})=\sqrt{\textbf{p}^2+m^2_{phys}}$.

If you want a very clear derivation for the fermionic version (spin 1/2) just look at Itzykson and Zuber's book on QFT. They also have a derivation for the photon propagator which I find really clunky yet is the only one I've found, having looked for it in about ten books on the subject.

This post imported from StackExchange Physics at 2016-05-31 07:23 (UTC), posted by SE-user Fra
answered Oct 1, 2015 by fra (155 points) [ no revision ]
I think this pretty much solves it. Im uncertain about a specific point, though: We take $P^\mu$ to be the exact momentum operator, and write 1-particle states as $P^\mu|p\rangle=p^\mu|p>$. Now, from first principles, how can we know that the eigenvalues satisfy $p^2=\text{const}$? In an interacting theory, we could expect to get corrections to the energy-momentum relation, in addition to mass renormalisation, right? If we want an axiomatic approach, we should prove somehow that 1-p states satisfy $(H^2-\boldsymbol P^2)|p\rangle\propto|p\rangle$. Is this possible?Is this important at all?

This post imported from StackExchange Physics at 2016-05-31 07:23 (UTC), posted by SE-user AccidentalFourierTransform
Also, in regards to the photon, I believe the Lorentz transformation properties of the fields are a bit more complicated that in the scalar field. If I recall, (see Weinberg I), massless vector fields transform as $A\to \Lambda A+\partial \Omega$, for $\Omega$ a scalar operator. Anyway, I'll try to tackle the general case (any sort of field), and if I get sensible results, Ill post them here for future reference. Bis dann, thank you very much, cheers :)

This post imported from StackExchange Physics at 2016-05-31 07:23 (UTC), posted by SE-user AccidentalFourierTransform
i've edited the answer to address some of these questions, hope it helps!

This post imported from StackExchange Physics at 2016-05-31 07:23 (UTC), posted by SE-user Fra
I wish I could upvote more that once :)

This post imported from StackExchange Physics at 2016-05-31 07:23 (UTC), posted by SE-user AccidentalFourierTransform
Hey... I hope im not annoying you... but... one last question please :) I think KL is very useful, eg, to prove that mass=first pole of the propagator... but in the case of massless particles, we get branch-cuts at $p^2=0$, so we cant say anything about Dyson resummation of the photon propagator: the true propagator has no pole at $p^2=0$, so that we don't really get any restriction on the photon self-energy, right? We cant say that a pole at $p^2=0$ in the propagator means that the photon remains massless in QED,right? does this question make sense to you?I dont know if I made myself clear...

This post imported from StackExchange Physics at 2016-05-31 07:23 (UTC), posted by SE-user AccidentalFourierTransform
The pole in $p^2=0$ might be isolated, the branch cut starts at $M^2_{th}$ at the threshold for the creation of multiparticle states. For istance the photon which is not self interacting will have the threshold at the mass of the electron thus in this case you can isolate the one particle contribution from the continuum. (pag 220 Itzkinson-Zuber) In general this is not true and you might not be able to isolate the one particle contribution. In the case of the photon the fact that it remains massless depends upon the fact that the $U(1)$ symmetry reamins unbroken. (i.e. gauge invariance)

This post imported from StackExchange Physics at 2016-05-31 07:23 (UTC), posted by SE-user Fra
you can say the symmetry keeps the pole where it is. Again in the case of $M^2_{th}=0$ there is a branch-point singularity and you can't separate the one particle state from the continuum.

This post imported from StackExchange Physics at 2016-05-31 07:23 (UTC), posted by SE-user Fra

Your answer

Please use answers only to (at least partly) answer questions. To comment, discuss, or ask for clarification, leave a comment instead.
To mask links under text, please type your text, highlight it, and click the "link" button. You can then enter your link URL.
Please consult the FAQ for as to how to format your post.
This is the answer box; if you want to write a comment instead, please use the 'add comment' button.
Live preview (may slow down editor)   Preview
Your name to display (optional):
Privacy: Your email address will only be used for sending these notifications.
Anti-spam verification:
If you are a human please identify the position of the character covered by the symbol $\varnothing$ in the following word:
p$\hbar$ysicsOve$\varnothing$flow
Then drag the red bullet below over the corresponding character of our banner. When you drop it there, the bullet changes to green (on slow internet connections after a few seconds).
Please complete the anti-spam verification




user contributions licensed under cc by-sa 3.0 with attribution required

Your rights
...